LSAT and Law School Admissions Forum

Get expert LSAT preparation and law school admissions advice from PowerScore Test Preparation.

 Adam Tyson
PowerScore Staff
  • PowerScore Staff
  • Posts: 5153
  • Joined: Apr 14, 2011
|
#39936
Exactly right, biskam - we have no way of knowing what might have happened under different circumstances. Even if the rise in standards had caused the shift in demographics among the student body we couldn't know that some other cause wouldn't have come along in its place (and of course we don't know anything about a causal relationship between those two things, so we are even further in the dark).

Nice job!
 megansayles7
  • Posts: 2
  • Joined: Apr 07, 2018
|
#44839
Is letter C not correct because it's not an inference and is instead stated directly in the stimulus? Because it is correct, so that's the answer that I incorrectly chose.

Thanks!
Megan
 Shannon Parker
PowerScore Staff
  • PowerScore Staff
  • Posts: 147
  • Joined: Jun 08, 2016
|
#44866
megansayles7 wrote:Is letter C not correct because it's not an inference and is instead stated directly in the stimulus? Because it is correct, so that's the answer that I incorrectly chose.

Thanks!
Megan
Answer choice C is incorrect because we do not know that it is true. Be careful when the stimulus deals in proportions or percentages and the answer choice deals in absolutes and vice-versa. We only know that the proportion of students that come from outside Markland has decreased, we do not know how this occurred. There are three ways that this could have happened; the total number of students could have decreased, with the bulk of the decrease coming from students outside of Markland, the total number could have increased with the bulk of the increase coming from students inside Markland, or the total number could have stayed the same with the number of students from Markland increasing and the number of students from outside of Markland decreasing. Thus, we do not have enough information to determine whether or not answer choice C is true, and thus it is the incorrect answer choice.

Hope this helps,
Shannon
 psalom625
  • Posts: 4
  • Joined: Oct 14, 2018
|
#59560
What does the bit "the academic standards at the college have risen" have to do with anything? Does that tie in to the second part of E where it says "tuition fees have increased over the past 10 years"? Are the testmakers correlating increasing academic standards with tuition fees?
 Malila Robinson
PowerScore Staff
  • PowerScore Staff
  • Posts: 296
  • Joined: Feb 01, 2018
|
#59596
Hi Psalom625,
To get to the correct answer (E) the part of the stimulus that you quoted is not needed. But it does tie in to a few of the incorrect answers (A, B & D) and is meant to draw people away from the correct answer.
Hope that helps!
-Malila
 hlee18
  • Posts: 25
  • Joined: Apr 10, 2019
|
#67529
Hello there,

I now understand why choice C is an answer choice that COULD BE TRUE, rather than MUST BE TRUE.

But I'm finding a weakness in choice E that is preventing me from reaching a full conciliation with this problem.

What we know:

Proportion of students (let's say Non-M for not from Markland) has decreased from equal to or greater than 66% of the student population to approximately 40% of the student population.

Choice E asks the reader to consider that the college's per capita revenue, otherwise known as total revenue divided by the number of students in order to get an average revenue per student, is constant after this change.

This can come from multiple ways, because, the decrease in proportion of Non-M students can come from different ways:

Way 1: Solely a decrease in Non-M students. Self explanatory.
Way 2: Increase in M students. Since the total number of students = M students + Non-M students, the proportion of Non-M students can decrease to approximately 40%, simply by admitting more M students.
Way 3: Both ways 1 and 2.

And this is where we see the problem. In order for Choice E to be must be true, one has to assume that Way 1 is the only way that the proportion of Non-M students decreased.

But let's suppose that it was actually Way 2. I believe this to be equally valid as Ways 1 and 3. Well, if this is the case, then tuition fees do not HAVE to increase. In fact, the reverse must be true. Yes, the M students pay half of what Non-M students pay. So if Non-M students pay 2x, then M students pay x. And if the proportion of Non-M students decrease, because the number of M students increase, then actually tuition fees must have decreased over the past 10 years, not increased.

The problem DOES NOT limit the total number of students to be constant.
 hlee18
  • Posts: 25
  • Joined: Apr 10, 2019
|
#68083
Just following up to see if anyone wishes to respond.
User avatar
 Dave Killoran
PowerScore Staff
  • PowerScore Staff
  • Posts: 5852
  • Joined: Mar 25, 2011
|
#68104
Hi hlee,

Just for everyone, I've added a complete explanation to this problem as the first post. You might find it helpful as you review this problem.

As for the scenario you pose, let's try an exercise: give me a numerical scenario where it all works according to the facts of the stimulus (similar to the one I use in the new first post but expanded to include tuition numbers per student origin) and to the scenario you've created, and still disproves (E) when the sufficient condition in that answer is activated. I think that seeing it numerically will help you see what's happening here (hint: I think you may have reversed the Markland and non-Markland tuition figures, or the proportional aspect somehow).

Thanks!
 SammyWu11201
  • Posts: 29
  • Joined: Jun 29, 2020
|
#77594
Hello. I don't understand why Answer Choice E is the correct answer. We know that the percentage of non-Marklanders has dropped, but that does not translate to an actual drop in the number of non-Marklander students. The overall total of the school could have increased in number, thus causing the share of the non-Marklander students to drop. Answer Choice E assumes that the drop in percentage of non-Marklander students translates to a drop in the number of them, and therefore if the college's per capita revenue from tuition has remained the same, tuition fees have increased over the past 10 years.
User avatar
 Dave Killoran
PowerScore Staff
  • PowerScore Staff
  • Posts: 5852
  • Joined: Mar 25, 2011
|
#77631
SammyWu11201 wrote:Hello. I don't understand why Answer Choice E is the correct answer. We know that the percentage of non-Marklanders has dropped, but that does not translate to an actual drop in the number of non-Marklander students. The overall total of the school could have increased in number, thus causing the share of the non-Marklander students to drop. Answer Choice E assumes that the drop in percentage of non-Marklander students translates to a drop in the number of them, and therefore if the college's per capita revenue from tuition has remained the same, tuition fees have increased over the past 10 years.
Hi Sammy,

Thanks for the question. First, this is a tough problem, but answer choice (E) is correct and does not make a faulty assumption. Second, if you get a chance, please go back to the first post on the prior page and take a look at the example I provided in the main explanation. The reason I mention that is that it agrees with a fair bit of what you've said here :-D For example:

You said:
SammyWu11201 wrote:We know that the percentage of non-Marklanders has dropped, but that does not translate to an actual drop in the number of non-Marklander students.
The explanation says:
  • "The fact that the non-Marklanders have dropped in percent does not mean that their actual number has decreased"

You said:
SammyWu11201 wrote:The overall total of the school could have increased in number, thus causing the share of the non-Marklander students to drop.
The example used actually shows an increase in the number of non-Marklander students.


I mention examples like this because it shows that as far as several of the facts, you are right on top of it and that's great! So, where did you run into issues? It's in your statement that "Answer Choice E assumes that the drop in percentage of non-Marklander students translates to a drop in the number of them, and therefore if the college's per capita revenue from tuition has remained the same, tuition fees have increased over the past 10 years." It doesn't, and the example on the prior page—which I'll copy here again—shows that is the false assumption they hope that students make:

..... ..... ..... ..... ..... ..... ..... ..... 10 years ago ..... ..... Today

Total number of students at Central ..... ..... 100 ..... ..... ..... 200
Markland

Number of non-Markland students ..... ..... 66 (66%) ..... ..... 80 (40%)
(people/percent of total)

Number of Markland students ..... ..... ..... 34 (34%) ..... ..... 120 (60%)
(people/percent of total)


So, I've addressed the very points you use in your concern in the main explanation, and then in the explanation for (E) further explained what is happening. If I've missed your point here, please let me know where. Otherwise, if this is still unconvincing, my challenge to you would be to come up with a numerical example that reveals the faulty assumption you think may have occurred, while addressing the per-capita revenue element which is key here.

Thanks!

Get the most out of your LSAT Prep Plus subscription.

Analyze and track your performance with our Testing and Analytics Package.